una serie particolare...

Il forum di Base5, dove è possibile postare problemi, quiz, indovinelli, rompicapo, enigmi e quant'altro riguardi la matematica ricreativa e oltre.

Moderatori: Gianfranco, Bruno

Rispondi
Admin
Amministratore del sito
Amministratore del sito
Messaggi: 870
Iscritto il: mer apr 20, 2005 3:47 pm
Località: Benevento

una serie particolare...

Messaggio da Admin »

Consideriamo la serie che si ottiene partendo dai due numeri $3$ e $7$ e tale che ogni termine ci è dato dal prodotto dei due precedenti, ossia:

$3+7+21+147+3087+...$

Quanto vale l' $n$-esimo termine della serie?
e il prodotto dei primi $n$ termini?
e la somma dei primi $n$ termini?

Ciao
Admin
Pietro Vitelli (Amministratore del Forum)
"Un matematico è una macchina che converte caffè in teoremi" Paul Erdös
www.pvitelli.net

elena
Nuovo utente
Nuovo utente
Messaggi: 7
Iscritto il: mer mag 25, 2005 3:09 pm

Messaggio da elena »

L' $n$-esimo termine della serie vale $3^{Fib(n-2)$$7^{Fib(n-1)}$, dove $Fib(n)$ è l' $n$-esimo termine della successione di Fibonacci.
Ne segue che il prodotto dei primi $n$ termini è fattibile. (non altrettanto facile per me scrivere la formula con tex).
Tutt'altro discorso mi sembra invece calcolare la somma dei primi $n$ termini.

ciao

elena
Nuovo utente
Nuovo utente
Messaggi: 7
Iscritto il: mer mag 25, 2005 3:09 pm

Messaggio da elena »

Provo con il prodotto dei primi $n$ termini.
Dovrebbe essere:
$3^{1+\frac{phi^{n-2}-(1-phi)^{n-2}}{\sqr{5}}$ $7^{\frac{phi^{n-1}-(1-phi)^{n-1}}{\sqr{5}}$, dove phi è il rapporto aureo $\frac{1+\sqr{5}}{2}$.
Per la cronaca ho utilizzato la formula di Binet, trovata su base cinque (manco a dirlo!) e segnalata tempo fa al forum da Tino, che fornisce la somma dei primi n termini della successione di Fibonacci.

Vediamo se si legge

ciao

Admin
Amministratore del sito
Amministratore del sito
Messaggi: 870
Iscritto il: mer apr 20, 2005 3:47 pm
Località: Benevento

Messaggio da Admin »

Ciao Elena,
penso sia giusta la tua soluzione riguardo al prodotto;
però non sono riuscito a semplificare gli esponenti nella tua sol.;
a me viene:
$\displaystyle\Large{ 3^{1+\left[\frac{\phi^{n-2}}{\sqr{5}}\right]}\cdot7^{\left[\frac{\phi^{n-1}}{\sqr{5}}\right]}}$

confesso che per la somma ho fatto dei tentativi, ma ho desistito subito.

idee per la somma?

Ciao
Admin
Pietro Vitelli (Amministratore del Forum)
"Un matematico è una macchina che converte caffè in teoremi" Paul Erdös
www.pvitelli.net

elena
Nuovo utente
Nuovo utente
Messaggi: 7
Iscritto il: mer mag 25, 2005 3:09 pm

Messaggio da elena »

Admin ha scritto:Ciao Elena,
penso sia giusta la tua soluzione riguardo al prodotto;
però non sono riuscito a semplificare gli esponenti nella tua sol.;
Perché ho scritto delle stupidate! No, non è giusta la mia souzione riguardo al prodotto. Ho preso un grossissimo abbaglio (diciamo così) riguardo alla formula di Binet. Non fornisce assolutamente la somma dei primi n termini della successione di Fibonacci (come da me scritto) ma solo l'n-esimo termine della successione, in modo non ricorsivo, ma soltanto in funzione di n. In pratica, se con $S(n)$ indichiamo la somma dei primi n termini della successione di Fibonacci, il prodotto dovrebbe diventare: $3^{1+S(n-2)}$ $7^{S(n-1)}$.
Siccome però $S(n-2)=F(n)-1$, si ha:

$3^{F(n)}$ $7^{F(n+1)-1}$, cioè:

$3^{\frac{phi^{n}-(1-phi)^{n}}{\sqr{5}}}$ $7^{\frac{phi^{n+1}-(1-phi)^{n+1}}{\sqr{5}}-1}$.

Urge controllo.

Per la somma sono in altissimo mare.

ciao
elena

Bruno
Livello 10
Livello 10
Messaggi: 2020
Iscritto il: lun nov 21, 2005 6:07 pm
Località: Bologna

Messaggio da Bruno »

...

Purtroppo devo correre e ho giusto il tempo di scrivere qualche riga.
Ok, credo che fin qui vada bene, Elena.
Riepilogo un attimo quello che è appena stato detto.
Sequenza proposta:

$t_{\small 0}=3,\;\; t_{\small 1}=7,\;\; t_{\small 2}=3\cdot 7,\;\; t_{\small 3}=3\cdot 7^{\small 2},\;\; t_{\small 4}=3^{\small 2}\cdot 7^{\small 3},\;\; t_{\small 5}=3^{\small 3}\cdot 7^{\small 5},\;\; t_{\small 6}=3^{\small 5}\cdot 7^{\small 8},\;\; \cdots$

Sequenza di Fibonacci:

$F_{\small (0)}=0,\;\; F_{\small (1)}=1,\;\; F_{\small (2)}=1,\;\; F_{\small (3)}=2,\;\; F_{\small (4)}=3,\;\; F_{\small (5)}=5,\;\; F_{\small (6)}=8,\;\; \cdots$

Troviamo allora:

. per n=0, $\,t_{\small 0}=3\,$,

. per n>0, $\,t_{\small n}=3^{\small F_{(n-1)}}\, \cdot7^{\small F_{(n)}} \,$,

dove:

$F_{\small (n)} = \frac{{\phi^{\small n}-(1-\phi)^{\small n}}}{2\phi-1}.$

Per il secondo membro, naturalmente, si conoscono forme abbreviate
finalizzate al calcolo pratico.

Il prodotto dei termini della successione fino a $\,t_{\small n}\,$, per il fatto
che gli esponenti si sommano, è:

$\prod_{i=0}^{n} t_i = 3\cdot 3^{\small {F_{(n+1)}-1}}\,\cdot7^{\small {F_{(n+2)}-1}}\, = 3^{\small F_{(n+1)}}\cdot 7^{\small {F_{(n+2)}-1}}$

ricordando questa nota relazione sulla somma dei primi termini della
sequenza di Fibonacci:

$\sum_{i=0}^{r-1} F_{\small {(i)}} = F_{\small {(r+1)}}-1.$

Ho provato anch'io qualche "divinazione" sulla somma, ma per ora
senza risultati.
Al momento sono piuttosto impegnato (per fortuna, vuol dire che c'è
lavoro), però spero di poter ritornare sul quesito di Pietro.

A presto :wink:
Ultima modifica di Bruno il ven mag 26, 2006 5:43 pm, modificato 1 volta in totale.
(Bruno)

...........................
Invisibile un vento
l'ha apena sfioragia
sospension d'un momento;
e la bola iridessente gera 'ndagia.
{Biagio Marin}
................................................................
Meglio soluzioni sbagliate che risposte esatte.
{Rudi Mathematici}

Admin
Amministratore del sito
Amministratore del sito
Messaggi: 870
Iscritto il: mer apr 20, 2005 3:47 pm
Località: Benevento

Messaggio da Admin »

Elena, in realtà l'abbaglio l'ho preso proprio io;
infatti ho scritto:
Admin ha scritto:a me viene:
$\displaystyle\Large{ 3^{1+\left[\frac{\phi^{n-2}}{\sqr{5}}\right]}\cdot7^{\left[\frac{\phi^{n-1}}{\sqr{5}}\right]}}$
ossia ho considerato il prodotto $3^{1+S(n-2)}\cdot7^{S(n-1)}$, solo che invece di sostituire ad $S(n-1)$ e $S(n-2)$ la somma dei primi $n-1$ e $n-2$ termini della serie di Fibonacci vi ho sostituito la formula di Binet che ci dà l'$n$-esimo termine della serie di Fibonacci.

Invece deve essere come hai scritto nel tuo ultimo post, ossia $S(n-2)=F(n)-1$ e quindi il prodotto è:

$3^{F(n)} 7^{F(n+1)-1}\quad\Rightarrow\quad 3^{\left[\frac{\phi^n}{\sqr5}\right]} 7^{\left[\frac{\phi^{n+1}}{\sqr5}\right]-1}$

dove $[]$ indica l'approssimazione all'intero più vicino;

mi sembra di capire (chiedo conferma) che

$\left[\frac{\phi^n}{\sqr5}\right]={\frac{\phi^{n}-(1-\phi)^{n}}{\sqr{5}}}$.

x Bruno
Bruno ha scritto: per n=0, $\,t_{\small 0}=3\,,$

. per n>0, $\,t_{\small n}=3^{\small F_{(n-1)}}\, \cdot7^{\small F_{(n)}} \,,$
ciò è corretto, però in tal caso se vogliamo ad es. il $58$-esimo termine della serie dobbiamo considerare $n=57$ e non $58$.
Lo stesso vale per il prodotto.

Admin
Pietro Vitelli (Amministratore del Forum)
"Un matematico è una macchina che converte caffè in teoremi" Paul Erdös
www.pvitelli.net

Bruno
Livello 10
Livello 10
Messaggi: 2020
Iscritto il: lun nov 21, 2005 6:07 pm
Località: Bologna

Messaggio da Bruno »

Pietro ha scritto:Elena, in realtà l'abbaglio l'ho preso proprio io (...)
...niente di male, l'abbaglio era comune: in realtà, tu non hai fatto altro che
acquisire il primo risultato proposto da Elena (brava comunque), migliorando
la forma degli esponenti per il calcolo :wink:

e ha scritto:(...) mi sembra di capire (chiedo conferma) che

$\left[\frac{\phi^n}{\sqr5}\right]={\frac{\phi^{n}-(1-\phi)^{n}}{\sqr{5}}}$

dove $[]$ indica l'approssimazione all'intero più vicino (...)
...confermo!
Infine ha scritto:x Bruno

(...) però in tal caso se vogliamo ad es. il $58$-esimo termine della serie dobbiamo
considerare $n=57$ e non $58$. Lo stesso vale per il prodotto.
...proprio così :D
(Bruno)

...........................
Invisibile un vento
l'ha apena sfioragia
sospension d'un momento;
e la bola iridessente gera 'ndagia.
{Biagio Marin}
................................................................
Meglio soluzioni sbagliate che risposte esatte.
{Rudi Mathematici}

elena
Nuovo utente
Nuovo utente
Messaggi: 7
Iscritto il: mer mag 25, 2005 3:09 pm

Messaggio da elena »

Bruno ha scritto:...
Il prodotto dei termini della successione fino a $\,t_{\small n}\,$, per il fatto
che gli esponenti si sommano, è:

$\prod_{i=0}^{n} t_i = 3\cdot 3^{\small {F_{(n+1)}-1}}\,\cdot7^{\small {F_{(n+2)}-1}}\, = 3^{\small F_{(n+1)}}\cdot 7^{\small {F_{(n+2)}-1}}$

A presto :wink:
Bruno, pensavo che i nostri prodotti differissero solo per una questione di notazioni, tu sei partito da $\,t_{\small 0}\,=3$ e io da $\,t_{\small 1}\,=3$.
Però Pietro chiedeva il prodotto dei primi n termini (da 1 a n oppure da 0 a n-1), non dei primi n+1 (da 0 a n).

Per la somma ancora niente, anzi sono pessimista.

Ciao
elena

Bruno
Livello 10
Livello 10
Messaggi: 2020
Iscritto il: lun nov 21, 2005 6:07 pm
Località: Bologna

Messaggio da Bruno »

...

Ok, Elena, l'avevamo già visto più sopra con Pietro: il prodotto dei
termini fino a $\,t_{\small n}\,$ è uguale all'espressione che tu hai appena riportato
(e il primo elemento della sequenza è appunto $\,t_{\small 0}\,$, così di solito iniziano
le sequenze numeriche).
Però $\,t_{\small n}\,$ è l' $\,n+1$-esimo termine e quindi il prodotto riferito all' $\,n$-esimo
termine (cioè $\,t_{\small n-1}\,$) è naturalmente questo:

$\prod_{i=0}^{n-1} t_i = 3^{\small F_{(n)}}\cdot 7^{\small {F_{(n+1)}-1}}\;.$

E' stato bene chiarir meglio la cosa: grazie Elena :D


(Bruno)
(Bruno)

...........................
Invisibile un vento
l'ha apena sfioragia
sospension d'un momento;
e la bola iridessente gera 'ndagia.
{Biagio Marin}
................................................................
Meglio soluzioni sbagliate che risposte esatte.
{Rudi Mathematici}

Rispondi